LSAT and Law School Admissions Forum

Get expert LSAT preparation and law school admissions advice from PowerScore Test Preparation.

 emilysnoddon
  • Posts: 64
  • Joined: Apr 22, 2016
|
#26320
I had trouble with this question. I was choosing between answer choices B and D. I had a hard time with answer choice D because I didnt feel as though it needed to be true that it would be reasonable for the city to adopt the new tourist plan. Just because they said it was reasonable to spend the money necessary to convince an automobile manufacture to build a plant there and the tourism plan is cheaper, couldnt it be true that there is some aspect of the tourism plan that makes it unreasonable while the automobile manufacture plan reasonable? I also see a flaw with B though that, just because it is reasonable in this case to convince a manufacturing company to build a plant doesnt mean that in general it is reasonable. Any guidance on this question would be much appreciated.

Thank you,

Emily
User avatar
 Jonathan Evans
PowerScore Staff
  • PowerScore Staff
  • Posts: 726
  • Joined: Jun 09, 2016
|
#26371
Emily, this is another good question and actually illustrates a fascinating nuance of Must Be True questions. The question stem for this problem reads "The city leader's statements, if true, provide the most support for which of the following?"

Here the key words are "provide the most support for." Contrast this wording with that of other Must Be True questions that ask "If the statements above are true, which of the following must also be true?" The slight difference in degree of emphasis actually makes a difference in questions such as these. In the former situation, that which we are discussing here, we are looking for something that is most "strongly supported." In other words, there will be direct explicit evidence backing up the correct answer choice. However, I often tell students that in these "strongly supported" supported situations, sometimes you will be able to come up with possible, though unlikely, confounding circumstances, as you have brought up here. These are kinda the "but what if the sun doesn't rise tomorrow" possibilities.

In the stimulus, the aspect of the two plans discussed vis-à-vis the effects of their adoption is how many jobs they will create. In the case of the automobile plant, regardless of how much money such a plant brings in, we know that spending a certain amount of money to bring it is "reasonable" because of the jobs that result. In other words, the reasonableness of the expenditure for the automobile plant is directly related to how many jobs it will create. Since we also know that the tourism plan will create just as many jobs as the automobile plant but cost less money to enact, the direct relationship between the amount of money spent and the amount of jobs created guarantees that adoption of the tourism plan is warranted.

Let me illustrate:

Given that the number of jobs created is the ultimate metric by which we measure whether a plan is reasonable, suppose the automobile plant costs

$100,000 but will bring in 10,000 jobs

now suppose the tourism plan costs

$5,000 but will bring in 10,000 jobs

Since the very fact that spending $100,000 to bring in 10,000 jobs IS reasonable, it must be true that spending $5,000 to achieve the same result is also reasonable.

There is direct evidence for (D) in the stimulus, so it is the credited response. I hope this explanation helps clear up your question. Please reply if you would like further explanation.
 emilysnoddon
  • Posts: 64
  • Joined: Apr 22, 2016
|
#26388
That example helps a lot -- thank you! It seems tricky - will have to keep my eye out for subtle differences in the question stem. Thanks again.
 nrpandolfo
  • Posts: 33
  • Joined: Feb 04, 2018
|
#45815
Is A out of scope? Also that it doesn't follow your example of being the ratio of reasonable cost to SAME amount of jobs that the auto plant or tourism increase would produce?
 nrpandolfo
  • Posts: 33
  • Joined: Feb 04, 2018
|
#45904
Hi,

Can we discuss why A is wrong?
 James Finch
PowerScore Staff
  • PowerScore Staff
  • Posts: 943
  • Joined: Sep 06, 2017
|
#45908
Hi nrpandolfo,

The issue with answer choice (A) is that it only deals with cost, but we don't know what the city's budget looks like or what sort of job-creation measures they could afford, if any. All we know is that the more expensive of the two job-creation options given, the automobile plant, is a reasonable investment. This would mean that the less expensive but equally effective option, the tourism plan, should also be a reasonable investment. In fact, the city may invest in both, or neither.

But the premises given in the stimulus don't by themselves lead us to conclude that cities should only invest in the least expensive options, only that when two options are equally effective, and the more expensive of the two is a reasonable investment, the less expensive option would also be a reasonable investment.

Hope this clears things up.
 deck1134
  • Posts: 160
  • Joined: Jun 11, 2018
|
#47319
If this were an assumption or justify question, could we conclude that answer choice A is accurate?

Thanks!
 Adam Tyson
PowerScore Staff
  • PowerScore Staff
  • Posts: 5153
  • Joined: Apr 14, 2011
|
#47439
I don't think so, deck. First, an assumption question would not work at all with this stimulus because the stimulus does not contain an argument! All the author has done is present us with some information, and he hasn't made any assumptions. Second, this stimulus says nothing about what should or shouldn't be done and makes no recommendations of any kind. So, he can't be making any assumptions about what the city should or shouldn't do!

The same goes for treating this as a justify the conclusion question. There is no conclusion to justify here, and answer choice A doesn't do anything to help the facts in the stimulus.
 1month2go
  • Posts: 7
  • Joined: Jul 04, 2018
|
#47966
I think the reason I got this question wrong was because I did not make the connection that spending the money on enticing the car manufacture was reasonable because of the jobs it would create. I felt like the stimulus did not make that relationship clear - I would've expected some sort of transition/connector phrase or word for example: Thus, it would be reasonable for blabla.

Any tips on how to avoid a similar error in the future?

Thanks!
 Adam Tyson
PowerScore Staff
  • PowerScore Staff
  • Posts: 5153
  • Joined: Apr 14, 2011
|
#48039
I don't think it really is a "thus" situation, 1month2go - that would be more of an argument than just a fact set. Instead, I think what we have here is four pieces of information:

1) the auto plant would create some jobs

2) the tourism plan would create as many jobs as the auto plant

3) the auto plant would be a reasonable choice

4) the tourism plan would cost less than the auto plant

From there, we are supposed to make an inference. What seems reasonable under the circumstances? If the auto plant is reasonable, and the tourism plan costs less, and they have at least one set of benefits that are the same, then it seems like perhaps the tourism plan would also be a reasonable choice.

This answer doesn't have to be true, of course, because there could be other pros and cons worth considering, but the stem is not a Must Be True question but its weaker sibling, a Most Strongly Supported question. That means that based solely on the facts we have, with no outside info or assumptions, we want the answer that seems to get the most support of the five choices. Think of it as being a reverse-Strengthen question, where the stimulus supports the answer choice. We don't have to prove it, but we do have to help it using only those facts that we were given.

When the stem asks what must be true or what is most strongly supported, don;t look for a conclusion in the stimulus. Instead, use the facts in the stimulus to draw your own conclusion!

Get the most out of your LSAT Prep Plus subscription.

Analyze and track your performance with our Testing and Analytics Package.